anyone pls help me in qn 6 pls

Anyone Pls Help Me In Qn 6 Pls

Answers

Answer 1

Answer:

Step-by-step explanation:

I don't know if this is what your looking for but the 38 cars that are 10 years old are 45.78 in percentage and the 5 cars that are more than 20 years are 6.02 in percentage.

(if you are asking how much more percentage to get from 6.02 to 45.78 its 39.76)

I really did not get the question but hope it help a bit.

Answer 2

Answer:

(A) 46% (B) 13%

Step-by-step explanation:

First we can find the percentage of cars that are at least 10 years old by dividing the amount that are 10+ years old by the total amount of cars

[tex]38/83= 0.45783[/tex]

when made into a percentage that is 46%

Now we can find the percentage of those cars that are 20+ years old by dividing the number of 20+ year old cars by our percentage

[tex]5/38=0.13157[/tex]

When made into a percentage That is 13%

Hope this helps! :)


Related Questions

Directions: Read and understand and following situations, then do what is asked.

1. Krista's is score in her Math quiz is 30. Des's score is 20 less than twice Krista's score.
Dodie's score is 4 less than Krista's score.

a. What is score score of Des? _________
b. What is the score of Dodie? _________
c. What is the average score of the three girls? __
d. Who scored the least? _______


2. Vincent's weight is 6 kilograms more than Ezekiel's weight Daniel's is 3 kilograms less than Ezekiel's weight. The average weight of the 3 boys is 63 kilograms.

a. What is the weight of Vincent? ________
b. What is the weight of Ezekiel? ________
c. What is the weight of Daniel? _________
d. What is the total weight of the three boys? ____



Need Answer!! A.S.A.P​

Answers

Formula to find average:

Total value / number of values

____________

1. Krista = 30

a) Des = 2(30) - 20 = 40

b) Dodie = 30 - 4 = 26

c) Average score = 40 + 30 + 26 / 3 = 32

d) Dodie scored the least marks.

_____________

2. Ezekiel = x

Vincent = 6 + x

Daniel = x - 3

Average = 63

63 = x + 6 + x + x - 3/3

63 × 3 = x + 6 + x + x - 3

189 = 3x + 3

189 - 3/3 = x

62 = x

a) Vincent = 6 + x = 68 kg

b) Ezekiel = x = 62 kg

c) Daniel = x - 3 = 59 kg

d) Total weight = 62 + 68 + 59 = 189 kg

_____________

Hope it helps ⚜

Drag the tiles with the correct pairs

Answers

The middle one first then the last one second then the first one first

A person distributed his property of Rs 1000000 among his three sons and one daughter. Each son got 27% of the property. How much did the daughter get?

[tex] \mathscr{HELP!!}[/tex]

Answers

Brainliest plss!!!
Have a good day :)

Answer:

Rs 190000

Step-by-step explanation:

Total property = Rs 1000000

For 1 son= 27% of the property

For 3 sons=27%×3= 81%

For daughter=100%-81%=19%

Now,

For daughter

19% of Rs 1000000

19/100 × Rs 100000

Rs 190000

Hence the daughter will get Rs 190000

You got 250 from your mom and your dad 250 and bide something for 450 and your friend takes 30 and your mom takes 10 and you dad takes 10 and everyone asks for there things back so you give your mom 240 and you dad 240 and your friend give you 30

Answers

Answer:

-450 dollars

[tex]500 - 450 = 50\\50 - 50 = 0\\0 - 480 = -480\\-480 + 30 = -450[/tex]

I hope this helps! Have a great day.

Answer:

-$450

Step-by-step explanation:

yup

I was reading a book of 560 pages. On Sunday, I was able to finish 1/4pages of the book. How many pages do I read on Sunday?

Answers

Answer:

140

Step-by-step explanation:

560. 1

____. ÷. ______

1. 4

Cross out the numbers that are divisible

560/4 = 140

Its an algebra solving distance rate time
A jet travels 4470 miles against a jetstream in 6 hours and 5610 miles with the jetstream in the same amount of time. What is the rate of the jet in still air and what is the rate of the jetstream?

Answers

super easy make sure to put 8 with the 4470 because not only does it help you out but you always aff lip

Which of the following inequalities best represents the graph above?

a > 9

a < 9

a _< 9

a _> 9

Answers

Answer:

C

Step-by-step explanation:

Because it can be 9 or less than 9

Answer:
The answer would be C

Step-by-step explanation:

The reason why the answer would be C is because if it were to only be exactly at nine it would be the symbol of just less then but because the circle is filled that means that is less then or equal to

Have a good day

What is the rectangular form of the polar equation?

Answers

Answer:

C

Step-by-step explanation:

r - rcosθ = 6

r - x = 6

r = 6 + x

r² = (6 + x)²

x² + y² = 36 + x² + 12x

y² = 36 + 12x

12x = y² - 36

x = y²/12 - 3

That is C

1 box = $ 0.054, how many boxes will the guy get if he pay $50?​

Answers

He will get 2,700 boxes

Answer:

He would get 30 boxes I think

Step-by-step explanation: I did the math, I hope its correct

What is the height of a cylinder the volume is 1385.44mi² and the diameter is 14mi?

Answers

Answer:

9 mi

Step-by-step explanation:

V = 1385.44 mi³, radius r = 14/2 = 7 mi, find h

Use V = πr²h

1385.44 = 3.14 * 7² * h = 153.86 h

so

h = 1385.44/153.86 = 9 mi

The product of 3 and x is negative 18.

Answers

Answer: -6

Step-by-step explanation:

we know that 3x6=18

But, if you make it -6,

It will be -18!

it is -6

hope it helped :)

A round race track has a radius of 75 yards. What is the circumference of the race track?

Answers

Answer:

471.24

Step-by-step explanation:

The formula for circumference is 2×pi×r. Replace the r with 75 and multiply.

circumference of the race track = 471.24 yards.

What is circle?

A circle is a two-dimensional figure formed by a set of points that are at a constant or at a fixed distance (radius) from a fixed point (center) on the plane. The fixed point is called the origin or center of the circle and the fixed distance of the points from the origin is called the radius.

Given,

radius of race track r = 75  yards

circumference of the park = 2πr

                                            = 2×3.1416×75

                                            = 471.24 yards

471.24 yards is the circumference of the race track.

Learn more about circle here:

https://brainly.com/question/29142813

#SPJ2

1. The diagram shown is two intersecting lines. The measure of <5is 47°.

(a) What is the measure of 27 ? How do you know. Explain your answer in complete sentences.
(b) Suppose the measure of 26 can be represented by (2x - 5). What equation can be written to solve
for the value of x?
(c) What is the value of x?

Answers

The measure of ∠7 is 47°. Angle 7 is a vertical angle to angle 5

Suppose the measure of ∠6 is represented represented by (2x - 5), the equation that can be use to solve for x is 42 + 2x = 180

The value of x is 69

Vertical opposite angles

Vertically opposite angles are angles that are opposite one another at a specific vertex and are created by two straight intersecting lines.

Vertical opposite angles are congruent. Therefore,

a. ∠5 ≅ ∠7 (vertical angles)

Therefore, angles 5 and 7 are congruent angles.

∠7 = 47°

b. Suppose ∠6 = 2x - 5 . let's find x

Therefore,

∠5 + ∠6 = 180(angle on a straight line)

The equation to solve the above is as follows:

47 + 2x - 5 = 180

42 + 2x = 180

2x = 180 - 42

2x = 138

x = 138 / 2

x = 69

learn more on intersecting lines here: https://brainly.com/question/24715845

Answer:

a)  <5 and <7 have congruent angles. That means that both angles are equal, they have the same measure. This means that the measure of <7 is 47°.

<7 = 47°

b) 5 and 6 are adjacent, so this is the equation that we have to do to solve for the value of x.

<5 + <6 = 180°

47° + (2x - 5)° = 180°

c) To solve the linear equation we need to isolate x.

47° + (2x - 5)° = 180°

(2x - 5)° = 180° - 47° = 133°

2x - 5 = 133

2x = 133 + 5 = 138

x = 138/2 = 69

QUESTION:-
Evaluate:
[tex] \rm{}a. \displaystyle{ \frac{3}{4} + \frac{5}{6} + \frac{7}{8} \: \: \: \: \: \: \: \: \: \: \: \: b.6 \frac{1}{3} + 2 \frac{5}{6} + 3 \frac{7}{9} }[/tex]
Don't spam​

Answers

Answer:

#a

[tex] \implies\displaystyle{ \frac{3}{4} + \frac{5}{6} + \frac{7}{8}}[/tex]

LCM of 4,6 and 8=2×2×3×2=24

Converting each of them into equivalent like fractions,

[tex] \implies{ \displaystyle \frac{3}{4} = \frac{3 \times 6}{4 \times 6} = \frac{18}{24} }[/tex]

[tex] \implies{ \displaystyle{ \frac{5}{6} = \frac{5 \times 4}{6 \times 4} = \frac{20}{24} }}[/tex]

[tex] \implies{ \displaystyle{ \frac{7}{8} = \frac{7 \times 3}{8 \times 3} = \frac{21}{24} }}[/tex]

[tex]{ \displaystyle{ \frac{3}{4} + \frac{5}{6} + \frac{7}{8} = \frac{18}{24} + \frac{20}{24} + \frac{21}{24} = \frac{59}{24} = 2\frac{11}{24} }}[/tex]

Shortcut method:-

[tex]\implies{ \displaystyle{ \frac{3}{4} + \frac{5}{6} + \frac{7}{8} }}[/tex]

[tex]\implies{ \displaystyle{ \frac{3 \times 6 + 5 \times 4 + 7 \times 3}{24} }}[/tex]

[tex]\implies{ \displaystyle{ \frac{18 + 20 + 21}{24} }}[/tex]

[tex]\implies{ \displaystyle{ \frac{59}{24} = 2 \frac{11}{24} }}[/tex]

Remarks:-

Whenever the answer is an improper fractions we should convert the answer into mixed number.Always reduce the answer to the lowest terms.You should try to do mental work and apply the shortcut method.

#b

[tex]\implies{ \displaystyle{6 \frac{1}{3} + 2 \frac{5}{6} + 3 \frac{7}{9} }}[/tex]

Convert to improper fractions,

[tex]\implies{ \displaystyle{ \frac{19}{3} + \frac{17}{6} + \frac{34}{9} }}[/tex]

LCM of 3,6,9=3×3×2=18

[tex]{ \displaystyle{ \frac{19}{3} + \frac{17}{6} + \frac{34}{9} = \frac{19 \times 6 + 17 \times 3 + 34 \times 2}{18} }}[/tex]

[tex]\implies{ \displaystyle{ \frac{144 + 51 + 68}{18} }}[/tex]

[tex]\implies{ \displaystyle{ \frac{233}{18} = 12 \frac{17}{18} }}[/tex]

Alternatively,

[tex]\implies{ \displaystyle{6 \frac{1}{3} + 2 \frac{5}{6} + 3 \frac{7}{9} }}[/tex]

[tex]\implies{ \displaystyle{(6 + 2 + 3) + \frac{1}{3} + \frac{5}{6} + \frac{7}{9} }}[/tex]

[tex]\implies{ \displaystyle{11 + \frac{1 \times 6 + 5 \times 3 + 7 \times 2}{18} }}[/tex]

[tex]\implies{ \displaystyle{11 + \frac{35}{18} }}[/tex]

[tex]\implies{ \displaystyle{11 + 1 \frac{17}{18} }}[/tex]

[tex]\implies{ \displaystyle{12 + \frac{17}{18} }}[/tex]

[tex]\implies{ \displaystyle{12 \frac{17}{18} }}[/tex]

#a

[tex] \large \sf \frac{3}{4} + \frac{5}{6} + \frac{7}{8} [/tex]

Write all numerators above the least common denominator 24

[tex] \large \sf \frac{3 \times 6 +5 \times 4 + 7 \times 3 }{24} [/tex]

Multiply the numbers

[tex] \large \sf \frac{18 +20 + 21 }{24} [/tex]

Add the numbers

[tex] \large \sf \frac{59 }{24} [/tex]

[tex]\red{ \rule{35pt}{2pt}} \orange{ \rule{35pt}{2pt}} \color{yellow}{ \rule{35pt} {2pt}} \green{ \rule{35pt} {2pt}} \blue{ \rule{35pt} {2pt}} \purple{ \rule{35pt} {2pt}}[/tex]

#b

[tex] \large \sf 6\frac{1}{3} + 2 \frac{5}{6} + 3 \frac{7}{9} [/tex]

Convert the mixed number to an improper fraction

[tex] \large \sf \frac{19}{3} + \frac{17}{6} + \frac{34}{9} [/tex]

Write all numerators above the least common denominator 18

[tex] \large \sf \frac{19 \times 6 + 17 \times 3 + 34 \times 2}{18} [/tex]

Multiply the numbers

[tex] \large \sf \frac{114+ 51 + 68}{18} [/tex]

Add the numbers

[tex] \large \sf \frac{233}{18} [/tex]

WHAT TIME IS A QUARTER TO SIX?

Answers

Answer:

05:45, 15 is a quarter of an hour.

Answer:

5:45 15 minutes is a quarter of an hour

Nine people go to a show. The adult tickets are $15 and the children tickets are $7. The total cost was $87, How many children and adults are there?​

Answers

Answer:

2 adults, 8 children.

Step-by-step explanation:

just add the prices together.

Find the cost of purchasing 30 roses. Show how you know.

Answers

Answer:

90 dollars

Step-by-step explanation:

so first you need to solve for the rate of change or slope with the equation

y 2 - y1 / x2 - x1

here i think the independent variable is number of roses since the dollars rely on the number of roses (sorry for my explanation its kinda bad)

so since number of roses is independent we know that it is x and that the price in dollars is y

anyways we have to substitute two values into the equation

y 2 - y1 / x2 - x1

(3, 9) (6, 18)

18 - 9 / 6 - 3

9 / 3

3

knowing that the rate of change is 3, to find the price for 30 roses all we have to do is:

let p be price

p = 3x

p = 3(30)

p = 90 dollars

you invest 5000$ into an account that is gaining 2.6% annual interest compounded continuously

how much will be in the account in 5 years

How much will be in the account in 10 years

Answers

Answer:5 years is 650 10 years is 1300

Step-by-step explanation:


A soccer team is playing a game in a stadium. The team fills a total of
592
592 seats on one side of the field and
238
238 seats on the other.

Answers

Answer:

anyone know the real answer please i need helpppppppp

Step-by-step explanation:

please

You are considering adding a new food product to your store for resale. You are certain that, in a month, the minimum demand for the product will be 6 units while the maximum demand will be 8 units. The estimated demand for this new product in any given month is 6 units (p=0.1), 7 units(p=0.4), and 8 units(p=0.5). If you choose to purchase 7 units for resale this month, what is the probability of selling all 7 units?

Answers

Answer:

Using EMV analysis, the number of units of the new product should be purchased for resale = Purchase 7.

The maximum EMV of profit you can make is 270.

Explanation:

We can use the following method to solve the given problem

Solution:

Using EMV analysis,

EMV (Purchase 6 for resale)= 6(40)(0.1) + 6(40)(0.4) + 6(40)(0.5)=240

EMV (Purchase 7 for resale) = [6(40)-60](0.1) +7(40)(0.4) + 7 (40)(0.5) = 270

EMV (Purchase 8 for resale) = [6(40)-2(60)] (0.1) + [7 (40) - 60] (0.4) + 8(40)(0.5)= 260

Largest EMV= 270; Choose to purchase 7 units for resale.

Step-by-step explanation:

Hopefully this helps :)

Larry wanted to buy a notebook computer. He had $700 to spend. The salesman tried to
sell Larry a computer that cost $850, but Larry said that was too expensive. The
salesperson said he could give Larry 25% off of the price if he used a coupon. Sales ta
is 9%. Could Larry afford the computer? How much would it cost?

Answers

Answer:

yes, $694.88

Step-by-step explanation:

You have to find the cost after applying the coupon and sales tax. First remove 25% from the price by finding 75% of the price. 850*.75=$637.5 after applying the coupon. Then you would have to increase it by 9% to account of sales tax, so multiply by 1.09. 637.5*1.09= $694.88 after sales tax. This means that Larry can very narrowly afford the computer.

4. The two trapezoids below are similar. What is the
length of EF?
24 cm
A
B
7.5 cm
G
H

F
UL
х
E
C
30 cm

Answers

It has to B A because always come first

Adele and her best friend, Valentina, want to rent a canoe to explore the lake by their campsite. The Boat Bunker charges $27 for 3 hours. Oar City charges $45 for 9 hours. Which store has the better value?

Answers

Answer:

Oar City

Step-by-step explanation:

boat bunker charges 9$ per hour

27÷3=9

while Oar City charges 5$ per hour

45÷9=5

How many points should the player lose for not rolling doubles in order to make this a fair game?
Three-fifths
5/8
5
10/6

Answers

Answer:

5 but fair losing would be three-fifths

Step-by-step explanation:

WILL MARK YOU BRAINLIEST!!
Ryan has collected 7 grasshoppers, 8 butterflies, and 9 katydids. Which
statement is NOT true?
The ratio of butterflies to katydids in Ryan's collection is 8:9.
For every 9 grasshoppers in Ryan's collection, there are 7 katydids.
There are 7 grasshoppers for every 8 butterflies.
The ratio of katydids to the total number of insects in Ryan's collection is 3 to
8.

Answers

Answer:

For every 9 grasshoppers in Ryan's collection, there are 7 katydids.

Step-by-step explanation:

87 children had to be taken to a basketball game. The school van could only fit 10 children in the van at one time. How many trips does the van need to make? A. 6
B. 7 C. 8

Answers

Answer:

(D) 9 Trips

87 / 10 = 8,7, so it takes at least 9 trips

can someone help me please

Answers

the answer to this question is x=E

If the function g(x) goes through the point
(-1,11), then the function -g(x) must go
through what point?
-
Help

Answers

Answer:

(-1,-11)

Step-by-step explanation:

Since the Transformation happened to the g(x)

where we multiplied g(x) by -1 and g(x) represents the y-value

so the y-value will be multiplied by -1

New y-value = 11*-1 = -11

so the New Point will be (-1,-11)


Think About a Plan
The gazebo in the photo is built in the shape of a regular octagon. Each side is 8 ft long, and the enclosed
area is 310.4 ft2. What is the length of the apothem?
• How can you draw a diagram to help you solve the problem?
• How can you use the area of a regular polygon formula?

Answers

Answer:

9.7 ft

Step-by-step explanation:

Use the formula for the area of a regular polygon with "n" sides with lengths "s" and apothem a:

A= 1/2 ans

1. Substitute A= 310.4

2. n= 8 (for octagon)

3. s = 8 (length of side)

310.4 = 1/2 (a)(8)(8)

310.4= 32a

a= 9.7 ft

Length of apothem: 9.7 ft

What is the solution of the equation?

3 5√ (x + 2) 3 + 3 = 27

explanation would be appreciated.

Answers

Answer:

x = (274/25)

Step-by-step explanation:

STEP 1:Isolate the square root on the left hand side

STEP 2:Eliminate the radical on the left hand side

STEP 3:Solve the linear equation

STEP 4:Check that the solution is correct

Original equation, root isolated, after tidy up     5√x+2 = 18     Plug in (274/25) for  x       5√(274/25)+2 = 18      Simplify      5√12.96 = 18      Solution checks !!     Solution is:       x = (274/25)

Answer:

that is if 35 at the beginning

x= -[tex]\frac{2386}{1225}[/tex]

and if it is 5

x=[tex]\frac{14}{25}[/tex]

Other Questions
You are given the graph of a linear function y = g(x) and another function with the equation (x) = 2x + 2. A. Which function has the greater rate of change and by how much? B. Which function has the greater y-intercept and by how much? C. Which function has the greater x-intercept and by how much? D. Which graph is steeper? Four friends went to the coast on vacationand wanted to split the trip's costs evenlybetween them. The house rental cost $400.The cleaning fee was $75 and they orderedpizza the first night for $25. How muchshould each friend contribute?A. $500B. $125C. $100D. $75 True or False? The filtrate - the material that is filtered from the blood - containswater, urea, glucose, salts, amino acids, proteins, and some vitamins. * Complete the description of what happens to a figure when the given sequence of transformations is applied to it: (x,y) (-x,y); (x,y) (0.6x, 0.6y);(x, y) = (x - 5, y + 5)Reflection over the (y-axis/x-axis/origin); (movement upwards/translation/transformation/dilation) with a scale factor of 0.6; translation 5 unit left and ____ units up. A 27-year-old pregnant woman at 16 weeks gestation presents with vaginal discomfort and discharge. Nucleic acid amplification is consistent with trichomonas. Gonorrhea and chlamydia testing is negative. What is the best treatment regimen Outline ways in which the productivity of land can be improved how many feet are equal to 168 inches. Please help (i guess cells will be under the biology) good evening! Can someone please answer this, ill give you brainliest and your earning 50 points. Would be very appreciated. Which is true? The number 100 is reduced by 30%. The number X is the result of that calculation increased by 30%. The number Y is equal to 100. Which statement is true? (x-1)(x+1)what is it equivalent to What is the mass of 5 moles of He (g) The process of making the organization into a well-ordered whole is referred to as Select one: A. motivating. B. planning. C. organizing. D. controlling. Which of the following DECREASED as an early result of the Industrial Revolution?Life expectancyBThe size of the middle classCThe length of the work dayDUrbanization 8 1/3--(1/2x+1/3)Simplify Microsleep is when you For a standard normal distribution,P(z < c) = 0. 3387Find c rounded to two decimal places Tara has two bags of M&Ms. The first bag contains 6 red, 5 blue, and 4 green M&Ms. The second bag contains 3 red, 2 blue, and 4 green M&Ms. Tara will select one M&M from each bag. What is the probability that Tara will select a blue M&M from each bag? Marlas mom bought a 36-pack of water bottles at the supermarket. Each bottle has 500 mL of water. How many liters of water is this?1,80036183,6000____________________________________________________________________8. The best unit of measure for the liquid in a tea cup is the liter.TrueFalse____________________________________________________________________10. Every week, Marlas mom buys 24 bottles of water that each have 16 oz. How many pints of water is this?32 cups12 cups16 cups24 pints What happens to the period of the motion when the spring constant increases? Does it increase, decrease, or stay the same?